Cardiac Physiology

¡Supera tus tareas y exámenes ahora con Quizwiz!

Preload and cardiac output

.An increase in EDV will cause an increase in ___________ and ____________.

Mild hemorrhage on Starling curve will cause stroke volume to..

A mild hemorrhage, which causes a decrease in blood volume and pressure, will evoke the baroreceptor reflex, which produces an increase in contractility and an increase in heart rate. The increase in heart rate indicates that the new stroke volume will be on the upper curve. The low blood volume and increased heart rate causes a decrease in end- diastolic volume.

Which of the following would result from a regurgitant aortic valve in a nonfailing heart? a. A decrease in diastolic pressure b. A decrease in cardiac energy consumption c. A systolic murmur d. A decrease in heart rate e. A decrease in systolic blood pressure

A. Blood leaks from the aorta into the left ventricle during diastole in patients with regurgitant aortic valves. The rapid flow of blood out of the aorta reduces diastolic pressure in the aorta. At the same time, the rapid flow of blood into the left ventri- cle causes an increase in preload, which results in a larger stroke volume. The large stroke volume injected into the aorta during systole would increase the volume in the arterial system during ejection and therefore increase systolic pressure. The increase in stroke volume would also be associated with an increase in energy consumption by the heart. Murmurs result from the turbulence of blood passing through a narrowed orifice, such as a stenotic valve or one which has not fully closed. Blood flowing from the aorta into the ventricle through the partially opened aortic valve produces the sounds responsible for the diastolic murmur associated with a regurgitant aortic valve. If too much of the stroke volume flows back into the heart during diastole, mean blood pressure will fall and the barorecep- tor reflex will cause an increase in heart rate.

Why does peripheral edema occur most often in the lower extremities? A. capillary hydrostatic pressure is higher in the feet B. plasma oncotic pressure is lower in the feet C. arterial blood pressure is higher in the feet D. peripheral resistance is lower in the feet

A. Due to gravity, the capillary hydrostatic pressure is greater in the lower extremities.

Which one of the following will be observed in a patient with aortic regurgitation? a. An increased ejection fraction b. An increased mean blood pressure c. An increased diastolic blood pressure d. A decreased pulse pressure e. A decreased left atrial end-diastolic pressure

A. In aortic regurgitation, some of the blood ejected during systole reenters the left ventricle during diastole. The retrograde flow of blood into the ventricle during diastole increases ventricular volume (preload). At the same time, the retrograde blood flow decreases the diastolic blood pressure. The low diastolic pres- sure allows the heart to produce a greater ejection fraction. The pulse pres- sure is widened because of the elevated systolic pressure (brought about by the increased stroke volume) and reduced diastolic pressure (as a conse- quence of backflow of blood through the incompetent aortic valve).

When flow through the mitral valve is restricted by mitral stenosis, a. Exercise can induce acute pulmonary edema b. Left ventricular preload increases c. Left atrial pressure diminishes d. Right ventricular end-diastolic pressure decreases e. Central venous pressure decreases

A. In patients with mitral steno- sis, the flow of blood from the left atrium to left ventricle during diastole is diminished. The left atrium progressively dilates and left atrial pressure rises. The increase in pressure is transmitted through the pulmonary circu- lation and eventually leads to an increase in right ventricular pressure. When cardiac output increases during exercise, restriction to flow through the mitral valve causes a sudden increase in pulmonary blood volume, and pulmonary venous and capillary pressures rise, which leads to transuda- tion of fluid into alveoli and pulmonary edema.

Positive inotropic drugs such a digitalis reduce ischemic cardiac pain (angina) in a dilated failing heart by a. Decreasing preload b. Increasing diastolic filling time c. Decreasing total peripheral resistance d. Increasing heart rate e. Increasing coronary blood flow

A. Ischemic cardiac pain is produced when oxygen demand is greater than the oxygen that can be delivered by the coronary arteries. Positive inotropic agents reduce oxygen demand by increasing cardiac contractility. Although under normal circumstances an increase in contractility will increase oxygen demand, in a dilated heart, it actually decreases oxygen demand. The increase in contractility allows the heart to generate adequate force without increasing preload as much. As a result, the volume of the heart is reduced. Reducing volume decreases wall stress, because, according to the law of Laplace, the wall stress is proportional to the product of force and radius (which is proportional to ventricular volume).

The upstroke of the SA nodal action potential is produced by opening a channel that is a. Primarily permeable to Na+ b. Primarily permeable to Ca2+ c. Primarily permeable to K+ d. Primarily permeable to Cl− e. Equally permeable to Na+ and K+

A. Opening the L-type calcium channel produces the upstroke of the SA nodal action potential. This is the same channel that is responsible for the plateau phase of the ventricular action potential. The SA nodal cells have very few of the Na+ channels that open during the upstroke of the ventricular action potential. And because the maximum diastolic potential of the SA nodal cells is approximately −60 mV, any sodium channels that may be expressed on their cell membrane will be inactivated.

Stroke volume is increased by a. A decrease in venous compliance b. An increase in afterload c. A decrease in contractility d. An increase in heart rate e. A decrease in coronary blood flow

A. Stroke volume depends on preload, afterload, and contractility. Decreasing venous compliance forces more blood into the ventricle, resulting in an increased preload and an increased stroke volume. Increasing afterload and decreasing contractility decreases stroke volume. An increase in heart rate may result in a decreased filling time and, therefore, a decrease in preload and stroke volume. A decrease in coronary blood flow causes a decrease in ventricular contractil- ity and stroke volume.

During a routine physical, a 54-year-old man is told that his arterial blood pressure is high. Which of the following changes to left ventricular blood volume would be greatest in this patient due to his high arterial blood pressure? A. increased end systolic volume B. decreased end systolic volume C. increased end diastolic volume D. decreased end diastolic volume

A. This patient has high blood pressure, which means high afterload. On the high afterload pressure volume loop, you can see that ESV is increased and all the other answer choices are either decreased or remain the same.

Your patient has a right atrial pressure and cardiac output represented by point B on the graph below. These are both greater than in a normal individual (point A). You want to return the right atrial pressure and cardiac output of your patient to normal, that is, to point A. An INCREASE in which of the following will help return your patient to a normal right atrial pressure and cardiac output? A. urine output B. cardiac output C. GI motility D. arteriole relaxation E. myocardial contractility

A. an increase in urine output will decrease blood volume, thus shifting the curve back towards normal.

Systolic pressure increases due to increase in MAP, diastolic pressure either does not change or it decreases. Pulse pressure would therefore increase due to increase in systolic pressure. TPR would decrease due to massive vasodilation in skeletal muscle.

An aerobically fit woman runs 5 miles on a hot August day in Dominica. What would you expect to happen to each of the following cardiovascular parameters during this period of increased activity?

Increase in afterload and venous compliance can cause stroke volume to..

An increase in afterload indicates that the new stroke volume will be on the lower curve. The increase in venous compliance results in a decrease in end-diastolic volume

Decrease

An increase in arterial resistance will ______________ transcapillary fluid filtration.

What effect would an increase in lower extremity skeletal muscle pump activity have on central venous pressure (CVP)?

An increase in skeletal muscle pump activity will drive volume from the legs towards the central veins, thus increasing the volume and pressure in that space.

What effect would an increase in peripheral venous compliance have on central venous pressure?

An increase in venous compliance will decrease CVP because an increases in venous compliance is like venous relaxation, which decreases pressure.

Which of the following changes will cause an increase in capillary interstitial fluid volume (edema)? A. arterial pressure deceases B. arterial resistance increases C. venous pressure decreases D. venous resistance increase

An increase in venous resistance will cause blood to "back-up" into the capillaries, thus increasing capillary hydrostatic pressure and increasing filtration. This can lead to peripheral edema.

A 66-year-old man presents at the emergency department with shortness of breath and mild chest pain with exertion, discomfort when lying flat, and waking up at night very short of breath. Upon auscultation, a diastolic murmur and mild pulmonary edema are detected. Cardiac catheterization reveals an increased left ventricular preload. What cardiac condition is he most likely suffering from?

Aortic Valve Regurgitation. Since the patient had a diastolic murmur, you can eliminate anything that does not produce a diastolic murmur from the list (Mitral valve regurgitation, aortic valve stenosis). Pulmonary edema indicates a left heart problem, so right heart problems can be eliminated (Tricuspid valve stenosis)

If this individual's pressure volume loop changes shape, such that it is narrowed, shifted to the right and the peak pressure is increased to 150mmHg, what is the most likely condition to have produced the change?

Aortic valve stenosis reduces stroke volume (narrow PV loop), and increases afterload (increased max pressure on PV loop). Left Ventricular Hypertrophy will result from aortic valve stenosis, but it will not be the cause of the changes to the PV loop.

C

As a result of Phase III of the Valsalva maneuver, heart rate increases due to: a. increased cardiac output caused by increased venous return. b. increased vasoconstriction caused by decreased atriopeptin release. c. baroreceptor reflex stimulated by low venous return & falling aortic pressure. d. increased peripheral resistance only.

When the radius of an arteriole is increased, which of the following vascular properties will be increased? A. blood viscosity B. capillary blood flow C. diastolic blood pressure D. hematocrit E. systolic blood pressure

B. According to the Poiseuille-Hagan equation, an increase in vessel radius will increase flow

Which of the following would cause a decrease in stroke volume, compared to normal resting value? A. stimulation of sympathetic nerves to the heart B. a heart rate of 230 beats per minute C. stimulation of the efferent vagus nerve to the heart D. increase in end diastolic volume E. reduction in afterload

B. At heart rates above 180-200 bpm, the systolic and diastolic periods are shortened so severely that SV decreases.

When the viscosity of blood is increased, which of the following will be decreased? A. blood pressure B. capillary blood flow C. radius of resistance vessels D. blood vessel length E. radius of capacitance vessels

B. Based on the Pouiselle-Hagen Equation, if you increase viscosity of blood, you will decrease blood flow.

The tendency for edema to occur will be increased by? A. Arteriolar constriction B. Increased venous pressure C. Increased plasma protein concentration D. Muscular activity

B. Increased venous pressure would increase Pc and increase filtration=edema

An increased preload would most likely be caused by an increase in a. Arteriolar tone b. Venous tone c. Myocardial contractility d. Heart rate e. Capillary permeability

B. Preload is the volume of blood within the ventricles at the end of diastole. Preload can be increased directly by increasing venous volume or decreasing venous compliance. Thus, an increase in venous tone would decrease venous compliance and lead to an increase in preload. Increasing capillary permeability would decrease vascular blood volume and lead to a decrease in preload. Indirectly, reducing cardiac output can increase preload. A reduction in cardiac output leaves a larger portion of the vascular blood volume on the venous side. The increase in venous blood volume leads to a larger preload. Cardiac output is increased by an increase in myocardial contractility and heart rate and so would tend to lower preload. An increase in arteriolar tone, leading to an increase in total peripheral resistance, would tend to trap more blood on the arterial side of the circulation and therefore lower the amount of venous blood. The decrease in venous vascular blood volume would lead to a decrease in preload.

Jim is a healthy 22 year old student who is taking running on a treadmill at his local gym. What hemodynamic responses to stroke volume (SV), heart rate (HR), and blood pressure would you expect to see as he begins exercising moderately? A. increase SV, increase HR, increase diastolic pressure B. increase SV, increase HR, decrease diastolic pressure C. decrease SV, increase HR, increase diastolic pressure D. increase SV, decrease HR, decrease diastolic pressure E. increase SV, decrease HR, decrease diastolic pressure

B. SV and HR increase due to more sympathetic stimulation, diastolic pressure drops (or remains the same) due to a drop in TPR.

A middle-aged man arrives at the emergency room after being stabbed during a bar fight. He has lost 500 ml of blood and his mean arterial pressure is 70mmHg. Due to this drastic decrease in arterial pressure, you would expect to be occuring in the brain? A. brain damage due to reduced cerebral blood flow B. relatively normal blood flow to the brain C. confusion and lethargy due to reduced cerebral blood flow D. multiple organ system failure due to reduced blood volume and flow

B. Since autoregulation maintains a relatively constant blood flow to the brain and other major organs between a mean arterial pressure of 60 mmHg to 180 mmHg, the blood flow to the brain will remain relatively normal in this patient since his MAP is 70 mmHg.

Stroke volume can be decreased by a. Increasing ventricular contractility b. Increasing heart rate c. Increasing central venous pressure d. Decreasing total peripheral resistance e. Decreasing systemic blood pressure

B. Stroke volume is deter- mined by preload, afterload, and contractility. Increasing heart rate decreases the time for filling during diastole and may decrease preload and therefore stroke volume. Increasing preload by increasing central venous pressure will increase stroke volume. Similarly, decreasing afterload by decreasing total peripheral resistance or systemic blood pressure will cause an increase in stroke volume. Increasing contractility will also increase stroke volume.

Which one of the following is the best index of afterload? a. Left ventricular end-diastolic pressure b. Left ventricular mean systolic pressure c. Pulmonary capillary wedge pressure d. Total peripheral resistance e. Mean arterial blood pressure

B. The afterload is the force that the sarcomeres must overcome in order to shorten during systole. According to the law of Laplace, this force is proportional to the pressure (P) and radius (r) of the ventricle during ejection (force ∝ P × r). The mean left ventricular systolic pressure would therefore be the best index of afterload.

Which one of the following characteristics is most similar in the systemic and pulmonary circulations? a. Stroke work b. Preload c. Afterload d. Peak systolic pressure e. Blood volume

B. The right and left ventricles are in series with one another so that the entire cardiac output (except for a small anatomic shunt) passes through both circulations. Since the two ventricles beat at the same rate, their stroke volumes are the same. However, the resistance of the pulmonary vasculature is much smaller than that of the systemic circulation; thus, the afterload and stroke work are greater on the left side than on the right side. Because the same cardiac output is ejected into a higher resistance, peak systolic pressure is higher on the left side than on the right side. Only about 10% of the blood volume is within the pulmonary circulation at any one time. About two-thirds of the blood volume is stored within the systemic veins and venules. Although the left and right preloads are not identical, they are very similar.

CO2 regulates blood flow to what organ?

Brain. It is autoregulated by PCO2. If metabolism increases or arterial pressure decreases the PCO2 will increase and cause cerebral vasodilation

Your patient has a right atrial pressure and cardiac output represented by point B on the graph below. These are both less than in a normal individual (point A). You want to return the right atrial pressure and cardiac output of your patient to normal, that is, to point A. Which of the following would help accomplish this? A. venous dilation B. a negative inotropic drug C. venous constriction D. a positive inotropic drug E. increased contractility

C. Point B on the graph indicates that the patient has either a decreased blood volume or an increased venous compliance, as compared to normal. So if you constrict the veins (answer chice C), this will decrease venous compliance and return the patient towards normal.

Stroke volume can be increased by a. Decreasing ventricular compliance b. Increasing venous compliance c. Decreasing total peripheral resistance d. Increasing heart rate e. Decreasing atrial contractility

C. Stroke volume is influ- enced by ventricular preload, afterload, and contractility. Decreasing total peripheral resistance may result in a decrease in afterload and therefore an increase in stroke volume. Decreasing ventricular compliance (making the heart stiffer) or increasing venous compliance will decrease ventricular filling and, therefore, preload and stroke volume. Similarly, increasing heart rate and deceasing atrial contractility will decrease filling and stroke volume.

During ventricular ejection, the pressure difference smallest in magnitude is between the? a. Pulmonary artery and left atrium b. Right ventricle and right atrium c. Left ventricle and aorta d. Left ventricle and left atrium e. Aorta and capillaries

C. The pressure gradient between regions of the cardiovascular system is directly proportional to the resistance of the intervening structures. During ventricular ejection, the aortic valves are open and do not offer any significant resistance to blood flow. Therefore, there is very little, if any, pressure difference between the left ventricle and the aorta. Since the tricuspid valve is closed during ven- tricular ejection, there is an appreciable pressure difference between the right ventricle and the left atrium, although this pressure difference is opposite in direction to the flow of blood through the circulatory system. Although pulmonary vascular resistance is relatively small compared with systemic vascular resistance, it nonetheless produces a pressure drop between the right ventricle and the left atrium. Since most of the resistance in the systemic vasculature occurs at the level of the arterioles, there is a large pressure gradient between the aorta and the capillaries.

Which one of the following values is greater in the pulmonary circulation than in the systemic circulation? a. The mean arterial pressure b. The arterial resistance c. The vascular compliance d. The blood flow e. The sympathetic tone

C. The same amount of blood flows through the pulmonary and systemic circulations. However, the mean blood pressure in the pulmonary circulation is much lower (15 5 L/minmmHg) compared to the mean blood pressure in the systemic circulation (90 mmHg). The pulmonary blood pressure is lower because the resistance to blood flow in the pulmonary circulation is much lower than the resistance to blood flow in the systemic circulation. In addition, the pulmonary blood vessels are more compliant than the systemic blood vessels. There- fore, although the stroke volume of the left and right ventricles is the same, the pulse pressure in the pulmonary artery (25/10) is lower than that in the aorta (120/80). The resistance of the systemic circulation is determined by the activity of the sympathetic nerves innervating the arteriolar smooth muscle. The smooth muscle surrounding the pulmonary vessels receives very little sympathetic innervation.

The figure below illustrates organ blood flow over a period of time. This is an example of which type of hyperemia, and what effect will this have on the local blood flow to the area following the hyperemic event? A. active hyperemia; increase B. active hyperemia; decrease C. reactive hyperemia; increase D. reactive hyperemia; decrease E. general hyperemia; increase F. general hyperemia; decrease

C. This is reactive hyperemia because of the acute occlusion, and the hyperemia will cause an increase in blood flow following removal of the occlusion.

During exercise, TPR decreases because of the effect of? A. The sympathetic nervous system on splanchnic arterioles B. The parasympathetic nervous system on skeletal muscle arterioles C. Local metabolites on skeletal muscle arterioles D. Local metabolites on cerebral arterioles E. Histamine on skeletal arterioles

C. local metabolites on skeletal arterioles which causes local vasodilation and decreased arteriolar resistance of the skeletal muscle

A, B and C are all correct because all of these choices increase myocardial oxygen consumption and thus myocardial blood flow via local metabolites.

Coronary blood flow will normal increase when A. Heart rate increase B. Sympathetic activity increases C. The heart is dilated

Which of the following would cause a decrease in blood flow through the vessels of a skeletal muscle? A. An increase of potassium in the surrounding extracellular fluid B. An increase of adenosine in the surrounding extracellular fluid C. Administration of alpha-1 receptor blocking drugs D. Administration of alpha-1 receptor stimulating drugs

D. Administration of alpha-1 receptor stimulating drugs. Because alpha receptor agonists (activators) will cause vasoconstriction, thus reducing blood flow. Increase in adenosine or potassium are incorrect because they are the predominant local metabolites responsible for dilation of blood vessels in skeletal muscle, thus increasing blood flow. Alpha receptor antagonists (blockers) is incorrect because they will prevent constriction of vessels, thus increasing blood flow.

A 57 year old woman presents complaining of light-headedness upon standing. Her blood pressure is 90/65 while standing versus 130/80 while in a supine position, thus documenting a significant case of orthostatic hypotension. Based on your knowledge of the baroreceptor response, which of the following would occur in a healthy individual during the act of changing body position from supine to standing (the initial effect of standing up)? Baroreceptor Firing Activity, activity in the NTS and Heart Rate? A) increase increase increase B) increase increase increase C) decrease increase increase D) decrease decrease increase E) decrease decrease decrease A. A B. B C. C D. D E. E

D. Decrease, decrease, increase In a healthy person, immediately upon standing the BP at the level of the heart will drop, thus initiating a baroreceptor response to low BP. The effect will be reduced baroreceptor firing, decreased NTS activity, and a resulting increase in heart rate.

Some types of medications used to treat hypertension work by producing a reduction in vascular smooth muscle tone. These medications relieve hypertension by reducing? A. stroke volume B. blood viscosity C. heart rate D. peripheral vascular resistance E. intracellular levels of cGMP

D. If you reduce vascular smooth muscle tone, the vessel will dilate, which will cause a reduction in resistance. As a result of the decreased resistance, the stroke volume will also be reduced, but this is a secondary result, and thus not the best answer.

Flow of fluid through the lymphatic vessels will be decreased if there is an increase in a. Capillary pressure b. Capillary permeability c. Interstitial protein concentration d. Capillary oncotic pressure e. Central venous pressure

D. Lymph flow is proportional to the amount of fluid filtered out of the capillaries. The amount of fluid filtered out of the capillaries depends on the Starling forces and capillary permeability. Increasing capillary oncotic pressure directly decreases filtration by increasing the hydrostatic (osmotic) force drawing water into the capillary. Increasing capillary pressure, capillary permeability, and interstitial protein concentration (oncotic pressure) all directly increase lymph flow. When venous pressure is increased, the capillary hydrostatic pressure increases and, again, capillary filtration increases. Lymph flow is approximately 2 to 3 L per day.

Propagation of the action potential through the heart is fastest in the a. SA node b. Atrial muscle c. AV node d. Purkinje fibers e. Ventricular muscle

D. The most rapid conduction of the action potential occurs through the Purkinje fibers. The slowest con- duction occurs in the AV node.

Correct statements about the increase in pulmonary blood flow during vigorous exercise include which of the following? a. The percentage of increase in flow is greater in the bases of the lungs than in the apices b. The increase in flow is caused by a greater-than-fivefold increase in pulmonary arterial pressure c. The increase in pulmonary blood flow is less than the increase in systemic blood flow d. The increase in pulmonary blood flow is accommodated by dilation of pulmonary arterioles and capillaries e. The increase in pulmonary blood flow is caused by sympathetic nerve stimulation of the pulmonary vasculature

D. The pulmonary circulation is a low-pressure system compared with the systemic circulation. Because of this low pressure and the hydrostatic pressure gradient from the top, or apex, of the lung to the bottom, or base, of the lung, the apex of the lung is not as well perfused as the base of the lung. During vigorous exercise, there is a large (up to sixfold) increase in cardiac output. The increased flow through the systemic circulation is equal to the increase in flow through the pulmonary circulation. Total lung flow increases and flow at the base of the lung is still greater than flow at the apex. However, the flow at the apex, since it was originally low, may increase by up to 800%, whereas flow in the base of the lung only increases by up to about 300%. The pulmonary vessels are very compliant. The increased flow causes recruitment of previously closed capillaries and dilation of pulmonary arterioles and capillaries. Because of this, pulmonary artery pressure normally does not increase or increases by only a small amount, and it rarely increases more than twofold.

Physiological causes of edema include:

Decrease arteriolar resistance will increase Pc, increase filtration and lead to edema Increase venous resistance will increase Pc, increase filtration and lead to edema Decrease plasma proteins will decrease capillary oncotic pressure, decreasing reabsorption and lead to edema Decrease in lymph drainage will increase Pi leading to edema

A

During Phase I of the Valsalva maneuver, increased aortic pressure is due to: a. increased intrathoracic pressure. b. increased vasoconstriction. c. decreased vagal stimulation of the heart. d. increased peripheral resistance.

D

During Phase II of the Valsalva maneuver, decreased blood pressure is due to: a. increased cardiac output. b. decreased cardiac output due to decreased heart rate. c. increased vagal stimulation of the heart. d. decreased venous return due to elevated intrathoracic pressure

B

During Phase IV of the Valsalva maneuver: a. venous return and cardiac output are decreased. b. venous return and cardiac output are increased. c. parasympathetic stimulation of the heart increases cardiac output. d. sympathetic stimulation of the heart decreases cardiac output.

The electrocardiogram is most effective in detecting a decrease in a. Ventricular contractility b. Mean blood pressure c. Total peripheral resistance d. Ejection fraction e. Coronary blood flow

E

A 16-year-old male died suddenly during high school basketball practice. He had recently complained of "always being tired" and said he sometimes felt like his heart was going to "beat out of his chest". An autopsy determined that he had right ventricular hypertrophy, and died of sudden cardiac death due to a right ventricular dilated myopathy. If this patient had a failing right ventricle (right heart failure), which of the following would most likely have been observed? A. decreased heart rate B. decreased right ventricular wall stress C. increased right ventricular ejection fraction D. decrease jugular venous pressure E. decreased pulmonary capillary hydrostatic pressure

E. A failing right heart cannot effectively pump blood to the lungs, therefore decreasing lung volume and decreasing capillary hydrostatic pressure there.

The increase in cardiac output from rest to severe exercise is chiefly due to an INCREASE in? A. blood viscosity B. ventilation/perfusion ratio C. peripheral resistance D. stroke volume E. heart rate

E. Stroke volume and heart rate are the two primary contributors to cardiac output. Stroke volume increased initially from rest to moderate exercise, then hits its maximal level. Heart rate increases from rest to moderate exercise, and continues to increase even after stroke volume is maxed out.

Filtration occurs when capillary fluid pressure is greater than plasma colloid osmotic pressure. Reabsorption occurs when capillary fluid pressure is less than plasma colloid osmotic pressure.

Filtration and reabsorption are both types of bulk flow. What are the primary determinants of these movements?

Location: carotid sinus and aortic arch. Stimulus: change in MAP

High pressure baroreceptor: list the location and stimulus

What would be the effect of reduced total peripheral resistance (TPR) on arterial pressure (MAP) if cardiac output remains at normal physiological level?

If TPR is reduced but CO is not changed, then MAP must be reduced.

Low

If a given pressure change produces a small volume change, the compliance of the vessel is said to be ____________________.

During which phase of the cardiac cycle is aortic pressure highest?

Immediately after the rapid ejection of blood during left ventricular systole, beginning of the reduced ventricular ejection phase

Interstitial colloid osmotic pressure will exert an osmotic force that will pull water (or maintain water) in the interstitial space.

In which direction will water flow in response to interstitial colloid osmotic pressure alone?

What increases filtration?

Increase in capillary hydrostatic pressure by: increasing arterial pressure, increasing venous pressure, arteriolar dilation (decreasing arteriolar resistance ), venous constriction (increase venous resistance).

A decrease in mean arterial pressure result in which of the following changes to vasomotor center activity and cardioinhibitory center activity?

Increase, Decrease. NTS activity is directly related to cardioinhibitory activity, but inversely related to vasomotor center activity. So if NTS activity goes down, cardioinhibitory center activity will go down and vasomotor center activity will go up.

Which of the following changes will cause an increase in myocardial oxygen consumption?

Increased size of the heart. Myocardial oxygen consumption is determined by the amount of tension developed by the heart. It increases with increased afterload, increase HR/SV or when the size of the heart is increased.

During which phase of the cardiac cycle is ventricular volume the lowest?

Isovolumetric ventricular relaxation

Location: vena cava, right atrium and pulmonary artery Stimulus: change in blood volume

Low pressure baroreceptor: list the location and stimulus

The figure below shows a normal normal PV loop (white) overlapped with a pathologic PV loop (red). What abnormal heart sound would you expect to hear when auscultating this patient?

None. The red loop is representative of mitral valve stenosis. You will hear no abnormal heart sounds for a patient with Mitral Stenosis because this valve pathology does not produce over-filling of the ventricle (S3) or reduced ventricular compliance (S4). You will hear S1 and S2, but they are normal heart sounds, and the questions asks for abnormal heart sounds.

During which phase of the ventricular action potential is the conductance to calcium highest?

Phase 2 the plateau where calcium enters the cell and triggers release of more calcium from the SR

Left atrium and pulmonary vein

Pulmonary wedge-pressure estimates the pressure in which structure?

At which site is the systolic blood pressure the highest?

Renal Artery. Pressure on the venous side of circulation (central vein-SVC,IVC, right atrium, renal vein) are lower than pressures on the arterial side. Pressure in the pulmonary artery is lower then its counterpart on the left side of the heart. In the systemic circulation systolic pressure is higher in the downstream arteries (renal) than in the aorta because of the reflection of pressure waves at branch points.

A 16-year-old female student is found to have an S3 heart sound and systolic murmur during and routine physical exam. What cardiac pathology is she most likely suffering from?

Since the patient had a systolic murmur, you can eliminate all diastolic murmurs from the list (Aortic Regurg and Mitral Stenosis). The S3 heart sound is produced from pathologies that cause a "volume overload" in the ventricle, which would be consistant with a Mitral Valve Regurgitation (Aortic stenosis produces an S4 heart sound from pressure overload).

Blood flow to which organ is controlled by sympathetic nervous system rather than by local metabolites?

Skin

Isovolumetric contraction

The QRS complex signals the beginning of ___________________

Which of the following conditions causes pulse pressure to increase? a. Tachycardia b. Hypertension c. Hemorrhage d. Aortic stenosis e. Heart failure

The answer is b. Pulse pressure is proportional to the amount of blood entering the aorta during systole and inversely proportional to aortic compliance. Pulse pressure increases with hypertension because hypertension causes aortic compliance to decrease. Whether the hypertension is a result of an increased cardiac output or an increased peripheral resistance, the higher arterial pressure is caused by an increase in arterial blood volume. The increased blood volume stretches the arterial wall, making it stiffer and decreasing its compliance. Stroke volume is decreased with tachycardia, hemorrhage, and heart failure, reducing pulse pressure in all three cases. In aortic stenosis, the ejection of blood from the ventricle is slowed and the increase in arterial blood volume during systole is less than normal.

Central venous pressure is increased by a. Decreasing blood volume b. Increasing venous compliance c. Increasing total peripheral resistance d. Decreasing heart rate e. Decreasing plasma aldosterone concentration

The answer is d Central venous pressure is the hydrostatic pressure in the great veins at their entrance to the right atrium. Increasing venous compliance would decrease pressure in the venous vessels and therefore would decrease central venous pressure. Decreasing blood volume would have the same effect. Reducing the plasma concentration of aldosterone would result in a decrease in blood volume. Increasing total peripheral resistance would tend to shift volume from the venous side of the circulation to the arterial side, resulting in a decrease in venous pressure. Increasing cardiac output tends to lower central venous pressure, whereas lowering cardiac output tends to increase central venous pressure. A reduction in heart rate would tend to lower cardiac output and therefore increase central venous pressure.

Cardiac output and Total Peripheral Resistance (MAP=CO*TPR)

The body keeps mean arterial pressure within a narrow range by regulating ____________ and _____________.

Connective tissue and smooth muscle cell

The compliance of a vessel is determined by the amount of ______________ and contractile activity of ___________________.

Hypertension and aortic stenosis

The following are examples of conditions that can increase afterload: _________ and __________________

The figure below is a cardiac-vascular function curve and the point marked control represents the state of the cardiovascular system in the resting state. An increase in contractility and arteriolar resistance is represented by a shift from the resting state to which new point?

The increase in contractility will shift the intersection point D, then the increase in arteriolar resistance will shift the intersection from point D to Point C. The second shift is due to the fact that most resistance lies in the arterioles, so if you increase resistance here, you will reduce cardiac output.

At which point on the above ventricular action potential is membrane potential most dependent on calcium permeability?

The plateau phase (phase 2) is the result of the influx of calcium. Although calcium channels begin to open during the upstroke (phase 0), the greatest number of calcium channels is open during the plateau. The upstroke is primarily dependent on the opening of Na channels. The initial repolarization (phase 1) is dependent on the inactivation of Na+ channels and the opening of a tran- sient K+ channel. Repolarization (phase 3) is produced by the inactivation of Ca2+ channels and the activation of the delayed rectifier K+ channels.

Diastolic blood pressure

The pressure at which the aortic valve opens corresponds to which arterial blood pressure measurement?

Individual diameter of a blood vessel

The primary determinant for velocity of blood flow is the __________________.

Plasma oncotic pressure and capillary hydrostatic pressure

The two major physiological determinants of transcapillary fluid movement are __________________ and ___________________.

Capillaries

The velocity of blood flow (cm/sec) in the ___________________ is slowest of all vascular structures.

TRUE OR FALSE. If cardiac output increases, causing an increase in mean arterial pressure and greater flow to an organ that can autoregulate (i.e. the brain), the resistance of the vessels supplying blood to that organ will increase.

True: Vessels will constrict (increase resistance) to prevent an increase in flow.

Which of the following is the result of an inward Na+ current?

Upstroke of the AP in Purkinje fibers, artia and ventricles

Calcium, preload

Ventricular contractility depends upon the intracellular concentration of _________ and is independent of _____________.

Wedge Pressure provides an estimate of the pressure in which part of the heart?

Wedge pressure is an estimate of the pressure in the pulmonary vein and the left atrium.

Colloid osmotic pressure is due to proteins and solutes in a given space. These compounds exert an osmotic pressure which tends to keep water in the compartment where they are located.

What is colloid osmotic pressure?

Mean arterial pressure is the average pressure in ALL Arterial vessels over the time spanning one single heart beat. MAP = Pdias + 1/3 (Psys - Pdias)

What is mean arterial pressure?

Arterioles regulate arterial pressure by changing arteriolar resistance.

What is the major function of the arterioles?

A. concentric hypertrophy is produced from conditions that cause a pressure overload in the ventricle, which is what we see in Aortic stenosis.

What type of hypertrophy will a chronic Aortic stenosis produce in the left ventricle? A) Concentric B) Eccentric C) None

When a person moves from supine position to a standing position what compensatory changes occur? A. Decrease heart rate B. Increase contractility C. Decreased TPR D. Decrease CO E. Increase PR interval

When this occurs blood pools in the leg veins causing a decrease in venous return to the heart, decreased CO and decreased arterial pressure. The baroreceptors detect the decreased in arterial pressure and the vasomotor center is activated to increase sympathetic outflow and decrease PNS. Increase in heart rate (results in a decreased PR interval), contractility and TPR. CO will return to normal.

D

Which of the following would cause a decrease in stroke volume, compared with the normal resting value? a. reduction in afterload b. increase in end diastolic volume c. stimulation of the efferent vagus nerve to the heart d. a heart rate of 230 beats per minute e. stimulation of sympathetic nerves to the heart

A and B are correct because they are the predominant local metabolites responsible for skeletal muscle dilation. C is correct because alpha receptor antagonists will prevent constriction of vessels, thus allowing more blood flow. D is NOT correct because sympathetic activation will cause vasoconstriction and reduce blood flow.

Which of the following would increase blood flow through a skeletal muscle? (choose all answers that are correct). a. An increase in tissue potassium b. An increase in tissue adenosine c. The presence of alpha receptor blocking drugs d. Sympathetic activation in the local active muscle

C

You have to determine the mean electrical axis in your patient quickly. This is what you know: The deflection in the QRS complex in lead I is net positive; the deflection in lead aVF is net negative. You report to your attending physician that the patient has: A. A normal mean electrical axis B. Right axis deviation C. Left axis deviation

Pulse pressure is...

determined by stroke volume. Pulse pressure is the difference between systolic and diastolic arterial pressures. It increases when capacitance of the arteries decreases (aging)

A patient is observed to have the following symptoms: reduced left ventricular end diastolic volume (LVEDV) associated with pulmonary edema, left atrial hypertrophy and a diastolic murmur. What is the valve defect that is consistent with these symptoms?

mitral stenosis: The diastolic murmur and reduce LV EDV make this the only correct answer choice.

What happens to local vascular resistance in the coronary arteries when shear stress increases at this location?

shear stress stimulates that production of nitric oxide, which causes vascular dilation and reduced resistance.


Conjuntos de estudio relacionados

Chapter 5: The circulation of the Oceans

View Set

chapter 5 understanding children from birth to age two

View Set

Oceanography: Chapter 7 Ocean Circulation

View Set

Life, Health, & Law Outline - KY State Law (Chapter 15)

View Set

Chapter 37: Management of Patients With HIV Infection and AIDS

View Set